If the only force acting on the player is gravity what is F in F=ma?

Answers

Answer 1

Answer:

  F is the force due to gravity

Step-by-step explanation:

F is the force resulting from gravity acting on the player whose mass is m. 'a' is the acceleration due to gravity, about 9.8 meters/second².


Related Questions

100 points and only highly intelligent persons are qualified

What is the equation for finding the value of √x. I need to be able to put x into the equation and result in the square root.​

Answers

You can solve it by using this steps

Step-by-step explanation:

Isolate the radical on one side of the equation.Square both sides of the equation.Solve the new equation.Check the answer. Some solutions obtained may not work in the original equation.

Answer:

It's probably 12

Step-by-step explanation:

100+100=200

200+100=300 divided by 10 is 12

Grace works as a salesperson at an electronics store and sells phones and phone
accessories, and makes a fixed commission for each phone sale and a different fixed
commission for each accessory sale. Yesterday Grace sold 18 phones and 21
accessories and made a total commission of $204. Today she sold 6 phones and 14
accessories and made a total commission of $82. Determine the price of each phone
and the price of each accessory.

Answers

We can use a system of linear equations to solve this. I will use the variable P for phones and A for accessories.

Our first equation is: 18P + 21A = 204. This is because she told 18 phones and 21 accessories yesterday which got her $204.

Our next equation is: 6P + 14A = 82. This is because she sold 6 phones and 14 accessories today which got her $82.

We can use elimination to solve this system. We can multiply the equation 6P + 14A = 82 by -3. This is so we can avoid sign errors if we multiply by 3. Our new equation is -18P - 42A = -246.

Next, we add that equation to 18P + 21A = 204. This gives us -21A = -42. Once we solve for A, by dividing by -21, we get A = 2.

Next, we can plug A into our equation. I will plug it into the second one. 6P + 14(2) = 82. This gives us 6P + 28 = 82. Once we get 6P alone, we get 6P = 54. Solving for P gives us 9.

One phone costs $9, and an accessory costs $2.

the sum of two opposite angles of a parallelogram is 140 degree . find all the angles ofa parallelogram?

Answers

Answer:

since opp. angles of a parallelogram is equal.

therefore,

let the angle be x

thus,

x+x=140

2x=140

x=70°

co interior angles are supplimentary,

thus,

x+y=180

y=180-70

y=110°

opp. angles are equal thus,

fourth angle =110°

1st angle=70

2nd angle=70

3rd angle =110°

Step-by-step explanation:

how would i plot this on a graph?

Answers

Answer:

(1, 4)

Step-by-step explanation:

im assuming that the point is h(1) = 4

this point is basically just h(x) = y

1 is your x

4 is your y

you plot it at the point (1, 4)

Ms. Rowlen went Black Friday shopping and she bought a flat screen TV that originally costs $259.00. She used her super savings coupon of 45%, how much did Ms. Rowlen pay for the tv?

Answers

Answer:

142.55

Step-by-step explanation:

An advantage of using graphics is the ability to
A) avoid data.
B) limit communication of information.
C) present information in a brief format.
D) There are no advantages to graphics.

Answers

Answer:

C

Step-by-step explanation:

Answer: C

Step-by-step explanation:

An advantage of using graphics is to present information briefly.

The LCM of 2, 3, and 4 is _____. 24 12 6

Answers

Answer: the answer is 12

Step-by-step explanation: trust me

The LCM of 2,3, and 4 is 12.

What is LCM?

The smallest number that can be divided by both numbers is known as the least common multiple (LCM) of two numbers.

The given numbers are 2,3 and 4.

The LCM of 2,3, and 4 is the lowest number that can be divisible by 2,3, and 4.

The lowest number that can be divisible by 2,3 and 4 is 12.

Therefore, the LCM of 2,3, and 4 is 12.

Learn more about LCM:
https://brainly.com/question/20739723

#SPJ2

Find the difference between 1/6 and 1/2

Answers

Answer:

1/3 OR -1/3

Step-by-step explanation:

1/2 - 1/6 = ?

3/6 - 1/6 = 2/6

2/6 = 1/3

OR

1/6 -1/2 =?

1/6 - 3/6 = -2/6

-2/6 = -1/3

Helpppppppppp plssssssssss

Answers

Answer:

12 in.

Step-by-step explanation:

We already know that the bottom line all together is 19 inches, and the line for the top of the square box at the end is also 7in. We can then infer that the top line of 7 cm is the same as the bottom line, so we therefore do 19 - 7 which equals to 12. Hope this helps.

A line passes through (9,2) and (12,-4). Write the equation of the line in standard form.
A. 2x + y = 20
B. 2x - y = 5
C. 2x - y = 16
D. 2x + y = 16

Answers

A line passes through (9,2) and (12,-4).So, x = 9, y = 2x = 12, y = -4

For the 1st equation,

2x + y = 20Putting x = 9, y = 2or, 2 (9) + 2 = 20or, 18 + 2 = 20or, LHS = RHSNow, putting x = 12, y = -4 in the above equation, we have2(12) + (-4) = 20or, 24 - 4 = 20or, LHS = RHS.So we can see that the first equation is the equation of the line.This is given in the graph in the picture.Like this way, if we check with other equations, we see that they are not the required equation of the line.

Answer:

A. 2x + y = 20

Hope you could get an idea from here.

Doubt clarification - use comment section.

BRAINLYEST PLUS 5 POINTS
PLEASE DO STEP BY STEP!:)

Evaluate 12x-3(3x-5)+24 when x = 2

Answers

Answer:

Step-by-step explanation: All we are doing here is inserting 2 in place of all the x variables. So that is written as 12(2)-3(3(2)-5)+24. Next, we follow the order of operations to solve

1) Multiply 3 by 2 and subtract by 5 inside the parenthesis = 12(2)-3-1+24

2) Multiply 12 by = 24-3+1+24

3) subtract 3 from 24 = 21+24

4) Add the final two terms = 45

MARK BRAINLIEST

(12)(2)-3((3)(2)-5)+24
=24-3((3)(2)-5)+24
=24-3(6-5)+24
=24-(3)(1)+24
=24-3+24
=21+24
=45
Sorry the parentheses got to be a lot LMMMMAAAAOOO anyways please mark as brainliest

PLEASE HELP ME WILL GIVE BRAINLIEST FOR RIGHT ANSWER​

Answers

Answer:

Step-by-step explanation:

I dont really know how to find out the variables but what i assume to do is this:

Find the square root of n and add it to m. The sum of both you divide by p. then when you figure that out, you distribute the x value into the parenthesis. This might be obvious but i looked at this question for 10 minutes and couldnt  find the value sorry :(

Answer:

m = 11
n = 145
p = -4

As you substitute the equation -2x^2 – 11x + 3 = 0 (from factorising x(-2x-11) = -3) into the quadratic formula to find the values of m, n and p.

26. A(-4, -1), B(-4, 6), C(2,6), D(2, -4)

Answers

Answer:

I NEEEEEEEED TO SEEEE GRAPHHHHHHHHHHHH

Step-by-step explanation:

Help plz, no links.

Answers

Answer:

1) g = -8 (one solution)

2) b = 0 (infinitely many solutions)

3) w = 5/6 (one solution)

4) c = 0 (infinitely many solutions)

Step-by-step explanation:

1) 1/2g -4 = 2g - 1/2g + 4

1/2g -2g +1/2 g = 8

-g = 8

2) 5.3-5.3 = 2.1b + b - 3.1b

0=0

3) 3/4w+ 3/4w = 10/4 - 5/4

3/2w = 5/4

w = (5/4) / (3/2)

w = 5/6

4) 5.7c + 3.2c - 7.8c - 1.1c = 1.5 - 1.5

0 = 0

what is the answer for this question 7(x+4)-7=48-2x

Answers

Answer:

3

Step-by-step explanation:

Expand

7x+28-7 = 48-2x

Simplify

7x+ 21 = 48-2x

Solve

Take 21 away from both sides

7x = 27-2x

Add 2x to both sides

9x = 27

Divide by 9 to find x

x = 3

How do I use Pythagorean Theorem to find out if a triangle has a right angle

Answers

In order to use the Pythagorean Theorem to find out if a triangle has a right angle, you have to determine If the squares of the two shorter sides add up to the square of the hypotenuse, then you know the triangle contains a right angle.

Write an inequality that represents the graph.

Answers

espaneool

Step-by-step explanation:

I need help with something in school.

Answers

Answer:

what do you need help with

Step-by-step explanation:

:)

Answer:

ask......

Step-by-step explanation:

What has a slope of -1/2 and an x intercept of -8

Answers

Answer:

y=(-1/2)(x)-8

Step-by-step explanation:

Since they give the slope and y intercept, you can literally plug it into the equation:

y=mx+b

where m is slope and b is y interecept.

Therefore, we get

y=(-1/2)(x)-8

4.3-3 (4r-3)=0.7 (6r-10)​

Answers

4.3-3(4r-3(=0.7(6r-10

We move all terms to the left:

4.3-3(4r-3(-(0.7(6r-10)=0

What are the solutions to
f (x) = x2 + 5x – 24

Answers

Answer: d/dx (f(x)) = 2x + 5
Ok done. Thank to me :>

Ryan, Alec and Amani went to ShopRite to buy boxes of fruit. Ryan bought 2 boxes of strawberries, 1 box of apples, and 4 boxes of blueberries for $34. Alec bought 3 boxes of strawberries, 2 boxes of apples, and 2 boxes of blueberries for $35. Amani bought 5 boxes of strawberries, 3 boxes of apples, and 2 boxes of blueberries for $49. How much does an individual box of strawberries, apples, and blueberries cost?

Answers

Answer:

5.

Step-by-step explanation:

First, we need to add all the number of baskets Ryan bought first. The equation should look something like this;

34÷(2+1+3)

Our answer is 4.8571428571. However, to put it in simplest terms, we can just say 4.85.

Now, we just keep changing up the numbers but with the same division we were using for Ryan's price on the fruit.

Alec's: 35÷(3+2+2)=5

Amani's: 49÷(5+3+2)=4.9

Notice how all the answers we get answers related to 5. If we all round the numbers, we actually get 5!

So, I believe your answer is 5. I'm terribly sorry if I'm incorrect. (If I am, I will try to fix it in the comments for you)


The range of the set of the values 7, 3, 6, 9 and 5 =

Answers

Answer:

6

Step-by-step explanation:

Since the range of a set is the difference between the highest and the lowest value in that set, the range here is:

9 - 3 = 6

Solve:

1. Square both sides of the equation.
2. Expand the left side. (3 − k)(3 − k) = 3k + 1
3. Multiply 9 − 6k + k2 = 3k + 1
4. Write the quadratic equation in standard form. k2 − 9k + 8 = 0
5. Factor the quadratic equation. (k − 8)(k − 1) = 0
6. Use the zero product property.
The solutions to the quadratic equation are
.

The true solution(s) to the radical equation

Answers

The solutions to the equation [tex]3-k=\sqrt{3k+1}[/tex] are k = 1 and 8

The given equation is:

[tex]3-k=\sqrt{3k+1}[/tex]

Square both sides of the equation

[tex](3-k)^2=3k+1[/tex]

Expand the left side of the equation above

[tex](3-k)^2=3k+1\\\\(3-k)(3-k)=3k+1\\\\3^2-3k-3k+k^2=3k+1\\\\9-6k+k^2=3k+1[/tex]

Write the quadratic equation in standard form

[tex]k^2-6k-3k+9-1=0\\\\k^2-9k+8=0\\\\[/tex]

Factor the quadratic equation

[tex](k-8)(k-1)=0[/tex]

Use the zero product property

k - 8 = 0

k = 8

k - 1 = 0

k = 1

The solutions to the equation [tex]3-k=\sqrt{3k+1}[/tex] are k = 1 and 8

Learn more here: https://brainly.com/question/25840704

answer

first one: 8 and 1

second one: is 1

Step-by-step explanation:

slay

Determine the period of the function y = -3cos(pi/5)x.

a. 3
b. 8
c. -3
d. 10

Answers

Answer:

Step-by-step explanation:

B

Answer:

10

Step-by-step explanation:

Rewrite the fraction 2/7 as a division expression with the same numbers.

Answers

Answer:

EZ. Btw you kinda already did it. 2/7 is a division problem. Another way is 2 divided by 7

Step-by-step explanation:


Tanaka says "When you multiply an odd number and an even number together, you will
always get an odd number'.
Show that Tanaka is wrong.

Answers

Answer: Tanaka is wrong becausee...

2 x 3 = 6

Since Tanaka said "When you multiply an odd number and an even number together, you will always get an odd number." we do not need to show more examples.

Hope this helps :)

If you received 5 one hundred dollar bills, 7 one thousand dollar bills and 7 one dollar bills, how much money did you get? Write the number in standard form.

Answers

Answer:

$7,507

Step-by-step explanation:

Reduce the radical 176

Answers

Answer:  [tex]\boldsymbol{4\sqrt{11}}\\\\[/tex]

Work Shown:

[tex]\sqrt{176}\\\\\sqrt{4*44}\\\\\sqrt{4}*\sqrt{44}\\\\2*\sqrt{4*11}\\\\2*\sqrt{4}*\sqrt{11}\\\\2*2*\sqrt{11}\\\\\boldsymbol{4\sqrt{11}}\\\\[/tex]

The idea is to factor the number such that we pull out perfect square factors. We use the rule that [tex]\sqrt{x*y} = \sqrt{x}*\sqrt{y}[/tex] to help break up the root. We also use the idea that [tex]\sqrt{x^2} = x[/tex] when x is nonnegative.

Matilda earns 48$ in 4 hours, How many hours does it take her to earn 288$?

Answers

Answer:

24

Step-by-step explanation:

If Matilda earns $48 in 4 hours, then she must earn 48/4 (12) dollars per hour. We know she earns $12 per hour, and that she made $288, so we divide the total amount earned (288) by her hourly wage ($12) to find she worked 288/12 or 24 hours.

Matilda will earn $288 in 24 hours.

What are word problems?

A word problem is a few sentences describing a 'real-life' scenario where a problem needs to be solved by way of a mathematical calculation.

Given is that Matilda earns $48 in 4 hours.

Let us assume that it will take her {x} hours to earn $288.

Then, we can write -

there is earning of $48 in 4 hours.

so, there will be earning of $1 ins (4/48) or (1/12) hours.

hence, she will earn $288 in (1/12 x 288) or 24 hours.

Therefore, Matilda will earn $288 in 24 hours.

To solve more questions on equation modelling, visit the link below -

brainly.com/question/29299318

#SPJ2

Other Questions
Lower chamber of Congress has how many members? would the person interviewing be considered a primary or secondary source? What can form along faults when pushed together or pulled apart?O MountainsO VolcanoesO Both Need help solving homework Pls help and show ur work ASAP Ill add extra points Julio is putting a rectangular pool in his backyard. The length ofthe pool is 7 feet more than the width. The perimeter of the poolis 54 feet. What is the length of the pool?23.5471710 De los tres cuartos de los asientos que hay en un camin, quitamos la tercera parte y aun quedan 20. Cuntos asientos tiene el camin?AYUDAA what is the chemical formula for Salt and hydrogen gas? how important is the 3 candle of advent? Explain the theme of FRIENDSHIP as it is carr-ied out in the novella. (Hint: Discuss Lennie and George and how their presence on the ranch changed the men) Of Mice and Men How does farming creates job for people? PLEASE HELP WITH THIS ONE QUESTION Which statement about oogenesis is correct?A) Diploid secondary oocytes undergo meiosis II to form a haploid ootid cell and a polar body.B) Haploid secondary oocytes undergo meiosis II to form a diploid ootid cell.C) Two diploid secondary oocytes undergo meiosis II to form four haploid ootid cells.D) Haploid secondary oocytes undergo meiosis I to form a haploid ootid cell and a polar body. [tex] \frac{3}{9} = \frac{?}{3} [/tex]What's the missing number Name two types of Crime Scene Sketches What characterizes the sender-receiver model of communication? A-The person who says something repeats it multiple times.B-The person who hears something consciously thinks about it.C-Only one person talks at a time.D-The person who hears something repeats what they heard back to the speaker Origins of Hinduism and Buddhism PPT ProjectDirections: You will create PowerPoint about the Origins of Hinduism and BuddhismFirst do your lessons and research: Complete your lesson and notes for Origins and Buddhism Suggested videos you may watch and use for your PowerPointBuddha and Ashoka: Crash Course World History #6Buddhism Explained: Religions in Global HistoryHinduism 101: Religions in Global HistorySecond create your PowerPoint. You will need 12 slides and include the following: CLICK HERETitle PageTable of ContentsSlides 3-5: Hinduism (include a fact and picture for each slide)Slides 6-8: Buddhism (include a fact and picture for each slide)Slide 9: Similarities of Hinduism and BuddhismSlide 10: Differences of Hinduism and BuddhismSlide 11: Location of both Hinduism and Buddhism (two maps)Slide 12: Bibliography (cite two sources) which conflict is most clearly a source of theodoras anxiety during the week that is described in the third paragraph as a long nightmare? A.She is letting her family down by neglecting her household duties.B.She is worried that her manuscript will be rejected by the publisher.C. The children have been having various problems she is ill equipped to handle.D.Mrs.Dave had been too sad and worried to help her care for her family. please help me guys thank you :) ! Angad was thinking of a number. Angad subtracts 5.1 from the number and gets an answer of 33.5. Form an equation with x from the information Solve for x: 2x + 12= 18 (5 points)